Best bid given the other's bid
by fancyfairy, Mar 30, 2025, 10:04 PM
Consider sets
(player) and
(items), with valuations given by matrix
![$
V = \left[\begin{matrix}
v_{1a} & v_{1b} & v_{1c} & v_{1d} \\
v_{2a} & v_{2b} & v_{2c} & v_{2d}
\end{matrix}\right]
$](//latex.artofproblemsolving.com/d/4/6/d465348852f12739e1fc2a04a9c2e5fdad0dbcb4.png)
where
.
A player’s total valuation is the sum of item values won, with half-value for half-won items.
Given player
's bid, player
's bid
is strictly better than another possible bid
of his if the total valuation derived from
is strictly more than that derived from
.
Each player simultaneously bids on items with a bid vector
such that
and
. The highest bidder wins an item; ties split ownership equally.
Given valuation matrix
, can we always find bid vectors
and
such that given
, the second player can not submit a bid strictly better than
and vice-versa?


![$
V = \left[\begin{matrix}
v_{1a} & v_{1b} & v_{1c} & v_{1d} \\
v_{2a} & v_{2b} & v_{2c} & v_{2d}
\end{matrix}\right]
$](http://latex.artofproblemsolving.com/d/4/6/d465348852f12739e1fc2a04a9c2e5fdad0dbcb4.png)
where
![$v_{ij} \in [0, \infty]$](http://latex.artofproblemsolving.com/7/8/8/788bffd362b46a318a2cb0703a9e2ae77089e4e7.png)
A player’s total valuation is the sum of item values won, with half-value for half-won items.
Given player






Each player simultaneously bids on items with a bid vector
![$B_i = [b^i_1, b^i_2, b^i_3, b^i_4]$](http://latex.artofproblemsolving.com/7/b/2/7b233ca3e9056aa374e937b1133cd9c81726f407.png)
![$b^i_j \in [0,1]$](http://latex.artofproblemsolving.com/b/9/6/b96c2a7618767934c8a44d9238c485fed39b8510.png)

Given valuation matrix





polynomial
by hanzo.ei, Mar 30, 2025, 3:12 PM
Given a polynomial
satisfying
,
, and for
(
) positive real numbers
. Prove that there exists a strictly increasing sequence of real numbers
such that
![\[
\sum_{i=1}^{n} \frac{k_i}{P'(a_i)} = \sum_{i=1}^{n} k_i.
\]](//latex.artofproblemsolving.com/4/4/e/44e5eb17b17531e36c362e062471d9bdffc8144c.png)







![\[
\sum_{i=1}^{n} \frac{k_i}{P'(a_i)} = \sum_{i=1}^{n} k_i.
\]](http://latex.artofproblemsolving.com/4/4/e/44e5eb17b17531e36c362e062471d9bdffc8144c.png)
VERY HARD MATH PROBLEM!
by slimshadyyy.3.60, Mar 29, 2025, 10:49 PM
Let a ≥b ≥c ≥0 be real numbers such that a^2 +b^2 +c^2 +abc = 4. Prove that
a+b+c+(√a−√c)^2 ≥3.
a+b+c+(√a−√c)^2 ≥3.
L
My problem
by hacbachvothuong, Mar 29, 2025, 10:10 AM
Bishops and permutations
by Assassino9931, Feb 29, 2024, 7:57 PM
Let
be a positive integer. Initially, a bishop is placed in each square of the top row of a 
chessboard; those bishops are numbered from
to
from left to right. A jump is a simultaneous move made by all bishops such that each bishop moves diagonally, in a straight line, some number of squares, and at the end of the jump, the bishops all stand in different squares of the same row.
Find the total number of permutations
of the numbers
with the following property: There exists a sequence of jumps such that all bishops end up on the bottom row arranged in the order
, from left to right.
Israel


chessboard; those bishops are numbered from


Find the total number of permutations



Israel
This post has been edited 1 time. Last edited by Assassino9931, Mar 4, 2024, 10:59 AM
Prove that AY is tangent to (AEF)
by geometry6, Aug 11, 2021, 9:03 AM
Let
be an arbitrary interior point of
, and
,
,
intersect
,
,
at
,
,
, respectively. Suppose that
be the midpoint of
,
and
intersect at
,
intersects
at
, and
intersects
at
. Show that
is tangent to
.
























IMO 2016 Problem 1
by quangminhltv99, Jul 11, 2016, 6:23 AM
Triangle
has a right angle at
. Let
be the point on line
such that
and
lies between
and
. Point
is chosen so that
and
is the bisector of
. Point
is chosen so that
and
is the bisector of
. Let
be the midpoint of
. Let
be the point such that
is a parallelogram. Prove that
and
are concurrent.






















This post has been edited 2 times. Last edited by sseraj, Jul 11, 2016, 4:41 PM
Prove that $\angle FAC = \angle EDB$
by micliva, Apr 18, 2013, 7:06 PM
Points
and
are given on side
of convex quadrilateral
(with
closer than
to
). It is known that
and
. Prove that
.
M. Smurov










M. Smurov
Archives





















Shouts
Submit
686 shouts
Contributors
aaaaarush • addyc • agoel2010 • aguathegreat • Alvie567 • Aminecraftbear • Beast_Academic_Emma • Bob_Smart • CC_chloe • chaihanrui • Cokevending56 • dbnl • DU4532 • em10 • enya_yurself • ericwzzhang • erringbubble • gamma_pi • goldenuni678 • Hestu_the_Bestu • hiloo • ICED • IcyBoba • IshiDe • ishipoopman • joshualiu315 • juicetin.kim • jxli.94 • linny.t • madeleinelee • math_sp • MrThinker • neha.s • Obuy • poplint0507 • potato_potato_potato • pupitrethebean • RandomeMids • SapphireDolphin9 • Seraph58750 • teasang • Technodoggo • ujulee • vanstheshoe • VRobot • WaterBeast
Tags
About Owner
- Posts: 4
- Joined: Jan 19, 2023
Blog Stats
- Blog created: Mar 31, 2023
- Total entries: 677
- Total visits: 24599
- Total comments: 2440
Search Blog